2
$\begingroup$

Let $M$ be a bounded hypersurface. Let $f \in H^{\frac 12}(M)$ and let $\varphi\colon M \to \mathbb{R}$ be a Lipschitz function.

When $M=\Omega \subset \mathbb{R}^n$ an open domain, we know that the multiplication $f\varphi \in H^{\frac 12}(\Omega)$. To see this, we can show the seminorm $|f\varphi|_{H^{\frac 12}}$ is bounded by adding and subtracting the same term, using triangle inequality and switching to polar coordinates.

Now let $M$ be the boundary of a bounded Lipschitz domain $\Omega \subset \mathbb{R}^{n}$, so $M$ is a compact bounded $(n-1)$-dimensional hypersurface. How do I show that $f\varphi \in H^{\frac 12}(M)$ too? I have trouble with the following term when following the same strategy as above: $$\int_M\int_M \frac{|f(x)|^2|\varphi(x)-\varphi(y)|^2}{|x-y|^{n}}d\sigma\leq \int_M|f(x)|^2\int_M |x-y|^{2-n}d\sigma$$ and I have no idea how to bound the term $\int_M |x-y|^{2-n}d\sigma$. Remember that $d\sigma$ is the surface measure.

What assumptions do I need to get this to work? Thanks.

$\endgroup$
2
  • $\begingroup$ Suppose that $M=\partial B$ where $B$ is the unit ball. Then you want to compute $\int_{\partial B} \frac{y \cdot n}{|x-y|^{n-2}} d\sigma{y}$ $\endgroup$
    – username
    Dec 29, 2013 at 15:50
  • $\begingroup$ You can get $$ \|fg\|_{H^{1/2}}\leq \|f\|_{C^1}\|g\|_{H^1} $$ by using interpolation of operators. You can see the paper by Gou & Tice, Analysis and pde, vol 6, n.2m 2013. $\endgroup$
    – guacho
    Jan 19, 2014 at 18:08

1 Answer 1

2
$\begingroup$

We have the bound $$ |fg|_{H^{0.5}}\leq C |f|_{H^{0.5+\delta}}|g|_{H^{0.5}}. $$ So, if you have a Lipschitz function, due to the compactness of the domain, the function is $H^1$.

$\endgroup$
2
  • $\begingroup$ Thanks, what is this bound called, because I want to look at its proof. $\endgroup$
    – soup
    Dec 28, 2013 at 13:17
  • $\begingroup$ Mmm, in the case $M=\mathbb{T}$ (or $M=\mathbb{R}$), the bound can be obtained using Kato-Ponce and the Sobolev embedding. $\endgroup$
    – guacho
    Dec 28, 2013 at 19:32

Your Answer

By clicking “Post Your Answer”, you agree to our terms of service and acknowledge you have read our privacy policy.

Not the answer you're looking for? Browse other questions tagged or ask your own question.